Go back to browse: Free exams and quizzes

Free Exam: DebopriyaRheumat301to400

Number of Questions in Test: 48
Number of Questions in Preview: 5
Register to view all questions.

Note: Answers are not shown below but will be copied with this test.

Copy this test to my quiz maker account

Register with ClassMarker to copy free tests to give to your Test takers.

Register now
Question 1
A 67-year-old man with poorly controlled seropositive rheumatoid arthritis for 22 years is followed up in Outpatients. He has noticed swelling around the eyes, especially first thing in the morning. Routine urinalysis reveals that he has+++ protein. His current medication includes methotrexate 20 mg weekly, sultasalazine 1g twice daily, prednisolone 5 mg once a day and ibuprofen 400 mg three times a day.



What is the likely diagnosis? 
Type: Multiple choice
Points: 4
Randomize answers: No
Question 2
A middle-aged woman with a history of rheumatoid arthritis presents with right foot drop and numbness in her right hand, with thenar eminence wasting and sensory loss over the radial half of her hand.



What is the most likely cause of her symptoms?  
Type: Multiple choice
Points: 4
Randomize answers: No
Question 3
25-year-old salesman complains he has had morning back pain and stiffness tor the last 5 months. He also complains of intermittent red and itchy eyes and arthralgia of his knees and elbows. He has a raised erythrocyte sedimentation rate (ESR) but the other blood test results are normal. A radiograph of the lumbar spine has been taken.



What is the most likely radiographic finding? 
Type: Multiple choice
Points: 4
Randomize answers: No
Question 4
A 22-year-old coal miner presents at the Emergency Department with exertional dyspnoea and wheezing, right foot drop, weakness in his left hand, a purpuric rash over his abdomen and swollen ankles. Blood tests show: haemoglobin 12.3 g/dl, mean corpuscular volume (MCV) 92 fl, white cell count (WCC) 21.4 x 109/1 (neutrophils 8.0 x 109/1, eosinophils 10.2 x 109/ 1, lymphocytes 2.5 x 109/1); urea 18.9 mmol/1, creatinine 270 μmol/1; cytoplasmic antineutrophil cytoplasmic antibody (cANCA) negative, perinuclear antineutrophil cytoplasmic antibody (pANCA) positive.



What is the most probable diagnosis? 
Type: Multiple choice
Points: 4
Randomize answers: No
Question 5
A 42-year-old woman with seropositive rheumatoid arthritis has become disabled by pain and ,tightness behind the right knee. Physical examination reveals a cystic swelling over the popliteal fossa and semimembranosus tendon.



Which one of the following is the most appropriate next step? 
Type: Multiple choice
Points: 4
Randomize answers: No
Register free

and create your first custom exams today - no credit card required.

Forgot password? / Register free